Ensino Médioprova sem intuição Tópico resolvido

Problemas sobre assuntos estudados no Ensino Médio devem ser postados aqui. Se o problema for de Vestibular, poste-o no fórum Pré-Vestibular

Moderador: [ Moderadores TTB ]

Autor do Tópico
Auto Excluído (ID:24530)
6 - Doutor
Última visita: 31-12-69
Jul 2020 07 09:48

prova sem intuição

Mensagem não lida por Auto Excluído (ID:24530) »

Seja a ≥ 0, prove que se a² ≤ 1 então a ≤ 1.
se a > 1 então a² > 1 (por contrapositiva)
fica claro que é verdade mas por quê? ha alguma maneira formal de explicar isso?

Última edição: Auto Excluído (ID:24530) (Ter 07 Jul, 2020 09:49). Total de 1 vez.



Avatar do usuário
csmarcelo
6 - Doutor
Mensagens: 5114
Registrado em: Sex 22 Jun, 2012 22:03
Última visita: 17-04-23
Jul 2020 07 10:08

Re: prova sem intuição

Mensagem não lida por csmarcelo »

É uma equivalência lógica. Isso é estudado em lógica.

Se [tex3]p\implies q[/tex3] , então [tex3]\sim q\implies\sim p[/tex3] .

Conhece esse assunto?




Autor do Tópico
Auto Excluído (ID:24530)
6 - Doutor
Última visita: 31-12-69
Jul 2020 07 10:10

Re: prova sem intuição

Mensagem não lida por Auto Excluído (ID:24530) »

csmarcelo escreveu:
Ter 07 Jul, 2020 10:08
Conhece esse assunto?
sim, o problema era para provar, ai eu usei a contrapositiva e cheguei em
se a > 1 então a² > 1
mas agora queria provar que isso é verdade de uma maneira rigorosa



Avatar do usuário
csmarcelo
6 - Doutor
Mensagens: 5114
Registrado em: Sex 22 Jun, 2012 22:03
Última visita: 17-04-23
Jul 2020 07 10:22

Re: prova sem intuição

Mensagem não lida por csmarcelo »

Ah tá... você quer provar que [tex3]a>1\implies a^2>1[/tex3] .

Cara, eu sou péssimo nesse tipo de questão, mas vamos lá... pensei o seguinte...

[tex3]a>1\implies a=\frac{m}{n},m>n[/tex3]

Assim,

[tex3]a^2=\frac{m^2}{n^2}[/tex3]

E

[tex3]m>n\implies m^2>n^2[/tex3] * isso é óbvio, não? :roll:

Logo,

[tex3]a^2>1[/tex3]



Avatar do usuário
snooplammer
4 - Sabe Tudo
Mensagens: 1701
Registrado em: Seg 24 Out, 2016 14:18
Última visita: 17-04-24
Jul 2020 07 12:05

Re: prova sem intuição

Mensagem não lida por snooplammer »

Suponha, por um momento, que [tex3]a > 1 \implies a^2<1[/tex3]

[tex3]a^2 - 1< 0[/tex3]
[tex3](a+1)(a-1)<0 \implies -1< a < 1[/tex3] , absurdo.

Última edição: snooplammer (Ter 07 Jul, 2020 12:06). Total de 1 vez.



Responder
  • Tópicos Semelhantes
    Respostas
    Exibições
    Última msg

Voltar para “Ensino Médio”